LSAT and Law School Admissions Forum

Get expert LSAT preparation and law school admissions advice from PowerScore Test Preparation.

User avatar
 Dave Killoran
PowerScore Staff
  • PowerScore Staff
  • Posts: 5853
  • Joined: Mar 25, 2011
|
#26559
Complete Question Explanation
(The complete setup for this game can be found here: lsat/viewtopic.php?t=11096)

The correct answer choice is (C)

If S is performed fourth, then from the last rule O cannot be performed fifth, and we can conclude that O must be performed first. This information eliminates answer choices (A), (B), and (E). Answer choice (D) can be eliminated because if P is second, and T is third, and we already know that S is fourth, then the placement of T violates the first rule. Thus, answer choice (C) is correct.
 lbayliyeva@unm.edu
  • Posts: 24
  • Joined: Jun 15, 2014
|
#15538
For question 16, how come answer choice C is correct?

I believe the correct answer looks like this: O P R S T F/L L/F H.

It makes sense that O has to be first because otherwise we will violate the last rule that says, "At least one composition is performed either after O and before S, or after S and before O." By that logic, answer choices A, B, and E get eliminated.

Answer choice D gets eliminated because then we will violate rule four that says, "The eighth composition performed is either L or H." If we select D as our answer choice, then we will have O P T S R _ _ F. We can have only either L or H eighth.

The only answer choice left is C, although it seems that there is a problem with it as well. Rule one says, "T is performed either immediately before F or immediately after R." If we choose answer choice C, then we will have O P R S L/H F L/H. We do not have T following immediately after R. It is for that reason that answer choice C seems to be wrong as well. Why is choice C considered to be the best/correct answer?

Thank you so much for your help in advance.
 Lucas Moreau
PowerScore Staff
  • PowerScore Staff
  • Posts: 216
  • Joined: Dec 13, 2012
|
#15556
Hello, lbayliyeva,

C is indeed the correct answer choice. The first four would have to be OPRS:

O P R S _ _ _ L/H

It could then look a few different ways after this:

O P R S T F L H
O P R S L T F H
O P R S T F H L
O P R S H T F L

While it is true that T is not following immediately after R, it is immediately before F in each of these four possible solutions. Does that make sense?

Hope that helps,
Lucas Moreau
 Holamariia
  • Posts: 1
  • Joined: Jan 09, 2018
|
#42773
Hello,

In this problem there are two conditions that say "at least two.." / "at least one.." - is it safe to say that there has to be AT LEAST two/one but could be more, as in three or two more spaces.. ? :-?
 Adam Tyson
PowerScore Staff
  • PowerScore Staff
  • Posts: 5153
  • Joined: Apr 14, 2011
|
#42792
Exactly right, and hola, Holamariia! "At least" means a minimum, just like in everyday use, so it allows for the possibility of more. That's a good thing to think about as you do your initial setup. We know that R and F cannot be any closer than 3 away from each other (two spaces between), but how far apart can you make them? Could they go, for example, first and last, with 6 things in between? Nope, because only L or H can be last. Maybe they can have 5 things in between? Try a hypothetical setup with F first and R 7th, or vice versa, and see if it works. What else would have to happen? Would that force you to violate any other rules? It's thinking like that that will often lead to key inferences.

Play around some, especially when doing untimed homework and drills, and you'll get a better feel for the games generally. That way, when you encounter something similar on another game in the future, you'll be better prepared to deal with it.

Good luck! Keep pounding!

Get the most out of your LSAT Prep Plus subscription.

Analyze and track your performance with our Testing and Analytics Package.